You are on page 1of 55

PAPER ll PART A STRUCTURED ESSAY

1. The apparatus used for determination of young, s modulus (Y) of a material in the form of a

wire in a laboratory consists of two identical vertical wires of the same material fastened to a rigid support. The wires carry a main scale (S) a vernier scale (V), a fixed load (W0) and a scale pan (P). (a). Draw a labeled diagram of this apparatus in the space provide. 7 cm available (b). What is the purpose of having two wires in this set- up. ( 2 lines ) (c ). In this experiment it is necessary to take readings while loading and unloding. Given reason. ( 2 lines ) (d). In such an experiment following measurements were taken by a student.

(1). Use the above set of reading to plot a graph of effective load ( W ) vs the corresponding increase in the length of the wire ( l ) , and find the gradient m. M= .

(2). What other additional measurement would yourequire to calculate Y for the material. State measuring instruments for the above measurements. Measurement (b) . (say ) Instrument

(3). In measuring one of the quantities mentioned in (2) above a particular procedure has to be followed in order to obtain an accurate value. State this procedure. (two lines )
(4). Write down an expressionfor the young, s modulus (Y) of the material used in this experiment interms of the gradient (m) of the graph and the measurements and .

Y = . (e). Young, s modulus of steel is approximately twice that of aluminium. Draw rough sketches of the stress ( S ) strain ( E ) curves for aluminium and steel onthe diagram given.

2. The method of mixtures is used in the laboratory to find the specific heat

capacity of lead in the form of lead shots. One of the major items used in this experiment is the calorimeter. (a). Make a list of other important apparatus used in this experiment. (two lines) (b). Statethe processes by which the heat is lost in the calorimeter. (two lines ) (c ). Outline briefly the methods adopted to minimize the heat loss form the calorimeter due to each initial temperature of the above processes. (5 lines) (d). What is the main reason to select a fixed temperature such as the boiling point of water as the intial temperature of lead shots? (two lines ) (e). State precautions that you would take when tranaferring lead shot to the calorimtter. (3 lines )

(f). In what way does the use of lead shots instead of large pieces of lead affect the value the final temperature of the mixture? Explain your answer. ( two lines ) (g). Cooling correction is important if the method of mixtures is used to determine the specific heat capacity of an insulating material such as rubber. Explain why.(2 lines) 3. An experimental set-up used in the laboratory to determine the speed of sound in air by resonating air columns is shown in the in the figure. (a). Briefly describe the procedure that you would adopt to obtain the fundamental note of vibration of the air column with this set-up. (3 lines) (b). If the corresponding length of the air column measured in (a) is and the wavelength of sound waves in air is , writ down the relationship between and . (neglect the end correction of the tube.) (one line) (C ) (1). Rewrite the expression in (b) interms of speed of sound in air V , and the frequency n of the tuning fork. (one lines) (2). If you are provided with several tuning forks of know frequencies and asked to determine V by plotting a graph state the quantities that you would plot Independent variable : Dependent variable : (d). with a certain tuning fork the value of was observed to be 3.5cm. If the length of the tube is 75cm, explain whether it is possible to find another position of the tube in water where resonance is produced with the same turning fork. (two lines) (e). If the room temperature is increased , would you expect to be greater than, equal to or less than 35cm. ? Give reason for your answer. (two lines) (f). If alcohol is used instead of water in this experiment, would you expect the same value for as measured in (C ) (2) ? Explain the answer. (two lines ) (g). For alcohol calculation has to be introduce only to the open end to the tube and not to the closed end, Explain why. (3 lines)

4. The diagram shown a slide- wire meter bridge. You are provide with a resistance box R, a coil of unknown resistance X, a slider S, a sensitive galvanometer G, a cell E, a key K, and some connecting wires.

(a). Draw, on the given diagram, the circuit you would use to determine the resistance X using the apparatus provided. (b). After connecting the circuit with a suitable value in R, and while attempting to find the balance point it is observed that the galvanometer deflection are always in the same direction. What may be the cause for this? (two lines) (c ) (1). For accurate determination of the unknown resistance what value in R is more suitable? (one lines) (2). Give the reason for your answer. (two lines) (d). In this type of experiment, while finding the balance point it is not advisable to slide S over the wire or press hard on the wire; Given the man reason for this. (tow lines) (e). In all meter bridge experiments, for the same values of X and R two balance lengths are usually obtained by having X and R interchanged in positions. Explain why. (3 lines) (f). In these experiment, it is advisable to use an additional resistance box R1 with the galvanometer, state the purpose of R1 ( two lines ) is most suitable than a meter bridge. Explain why. (4 lines)

PAPER ll PART B - ESSAY


1.Answer either part (a) or part (b).

(a). State Newton, s of universal gravitation. Two identical blocks A and B of mass 100 Kg each are placed on a light rough Horizontal surface in a region where there are no other objects except the earth Which exerts an attractive force on A and B in the vertically downward direction. The centres of gravity of the blocks are 0.1 m apart as shown in the figure and the Coefficient of static frietion of the horizontal surface is 0.1. (1). Taking the universal gravitational constant (G) to be equal to 6.67 x 10-11 Nm2 Kg-2 Find the magnitude of the gravitational force acting on B due to A. Indicate the Direction of this force on B. (2). Does the block B move towards A due to the gravitational force mentioned in (1)? Explain your answer. (3). Is there a frictional force acting on B?. If so, what is its magnitude? (4). Calculate the minimum mass that the block A should have in order to set the B in motion. (b). Define surface tension, Describe briefly a laboratory method to determine the surface tension of water using capillary rise. A glass tube of internal radius 12mm, wall thickness 0.4 mm, and open at both ends, is suspended vertically from a sensitive spring balance; A beaker containing a liquid is now brought slowly so that the surface of the liquid just touches the lover end of the suspended glass tube. What happens to the reading of the balance ? Explain your answer. The beaker of liquid is then raised until the original reading is seen again on the balance. If the deoth of immersion of the tube is 3.67 cm, calculate the surface tension of the liquid

assuming that the angel of contact of the liquid with glass is zero. Density of liquid is 1 000 Kg m-3. 2.State Archimede, s principle. A large thin walled cylindrical vessel fitted a mechanically operated pistion (p) at its open end as shown in the figure, is used to send a person to the sea bed to collect spenimens. The water level inside the vessel can be adjusted by raising or lowering the piston. The air pressure inside the vessel is always maintained at the atmospheric pressure by means of an internal air pumping system.

(1). When the vessel is put into the sea volume of air tapped inside the vessel is found to be 2 m3 and the vessel in found to float with 1/ 10th of this air volume above the sea level as shown in the figure. Find the mass of the vessel and its contents. (Density of water = 1 000 Km m-3) (2). If the area of cross section of the piston is 0.75 m2 find at least by how much the water level inside the vessel be raised in order to make it sink. (3). When the spccimens were stored in the vessel at the sea bed it was found that at least 0.05 m3 of water had to be removed from the vessel to lift off. Calculatc the mass of the specimens collected. (4). If the sea is 500 m deep how much minimum work has to be done on the piston in order to move the vessel up to the surface? Neglect viscous effects. 3. When an immersion heater is switched on without inserting it in water it will burn out soon. But when it is in water it works normally. Explain this statement.

What will be the maximum temperature attained by the surface of the heater when it is used to boil water? One end of a uniform lagged steel bar, 1 m long and area of cross section 0.01 m2 is kept at 100 0C and the other end at 00C as shown in the figure. A thin slot is cut across the bar 0.25 m away from the 100C end, and a 200 W flat electrical heating element is inserted into the slot as shown in the figure. The slot is then filled with mercury to ensure a good thermal contact between the heating element and the bar. When the heating element is switched on and the system has attained the steady condition calculate the surface temperature of the element Assume that mercury is a perfect conductor of heat. (Thermal conductivity of steel = 50 J m-1 S-1 k-1)

4.Describe briefly the wet- and dry bulb hygrometer and discuss how you would use it to find the relative humidity of the atmosphere. State precaution, if any that have to be taken in order to obtain an accurate value for the relative humidity. On a bay when the relative humidity of the atmosphere is 80% a certain room of capacity 48 m3 was isolated from the rest of the atmosphere and its relative humidity was reduced to 50% by means of a machine which absorbs water vapour from air without changing its temperature. Once the room had attained the room. Use the above data and the table to calculate (1). The dew point of the room to the nearest degree. (2). Absolute humidity of the atmosphere outside the room.
5. Answer either part (a) or (b) (a). What do you mean by dispersion of light? Define the dispersive power of a prism. Write down an expression

for the deviation of a monochromatic ray of light refracted through a small angled

prism. Two small angled prisms, made of crown (C ) and flint (F) glasses of dispersive powers 1 and 2 respectively, for the yellow colour , are placed as shown in the figure. If the prism combination is achromatic, show that, 1 (1 -1) A1 + 2 (2 - 1) A2 = 0 Where 1 and 2 are the refractive indicies of crown glass and flint glass respectively, for yellow light, and A1 and A2 are the respective refracting angles of the prisms. A flint glass prism is to be combined with a crown glass prism of refracting angle 100 so the combination is achromatic between the red and blue light. What should be the angle of the flint glass prism. Refractive index of flint glass for blue light Refractive index of flint glass for red light Refractive index of crown glass for blue light Refractive index of crown glass for red light = 1.6691 = 1.6501 = 1.5232 = 1.5146

(b). A compound microscope in normal adjustment is used to view a point object situated off the axis. Draw the paths of two light rays from the object to the eye through the microscope. Define the magnifying power of a microscope and explain why a compound microscope is usually employed rather than a single lens, when a large magnifying power is required. The desired overall magnifying power of a compound microscope is 140. The objective itself has magnifying power of 12 Find the required focal length of the eye piece assuming that the aaa image will be formed 25 cm from the eye. Derive any formula you may use. In a certain experiment it is necessary to place a cross writ in the compound microscope. aa on a diagram where this would be located. Assume that the final image is formed at infinity in this case. 6. Write down expressions for transverse and longitudinal wave speeds along a stretched wire shown that the expression for longitudinal wave speed is dimensionally correct. A heavy wire is hung freely and vertically from a fixed support. Transverse and longitudional waves are sent up long the wire , separatcly from

its lower end. Will these waves have constant speeds along the b wire? Explain your answer. A uniform steel wire of area of cross section 1.2 x 10-6 m2 is stretched horizontally. What should be the tension of the wire such that the speed of transverse wave along the wire is same as that of longitudinal wave? Explain why this condition cannot be achieved practically. Young s modulus of steel = 2 x 1011 Nm-2 m-3 7. Answer either part (a) or part (b) (a). Define the lerm resistivity of a conducting material. A d c generator supplies a voltage of 240 V to a resistive load of 120 situated 1 Km away from the generator. (1). If the resistive lode is connected to the generator by means of copper having diameter of 0.5 mm calculate the voltage of the d c generator. (Resistivity of copper = 1.7 x 10-8 m) (2). What is the power dissipated in the wires? (3). In the d c generator can provide only 241 V how would you supply the above mentioned voltage ( i.e 240 V) to the resistive lode using the wires of same material? (4). In long distance power transmission why it is advantageous alternating high voltage? to use an Density of steel = 7.8 x Kg

(b). Draw a clear labeled diagram showing the essential features of a moving coli galvanometer. Explain how a steady defection is produced when a steady current is passed through such a galvanometer. A galvanometer with a resistance of 39.8 is fitted with a shunt resistance of 0.2 to function as an ammeter with a full scale deflection of 10 A. What is the actual current through the galvanometer when is shown a full scale defection? This galvanometer is now to be used as a voltmeter with two different ranges having full scale defections of 3 V and 15 V respectively. What resistances should be used and how should they be fitted to achieve this? 8. State the laws of electromagnetic induction.

As shown in the figure a thin rigid wire is formed into a circular loop ABC and placed

perpendicular to a uniform magnetic field which is directed into the plane of the paper. The loop is connected to a resistance of 100 using thin connecting wires. The radius of the loop is 7 cm and the, magnetic flux density of the filed decreases with time at a constant rate of 10-2T s-1

(1). Neglecting the effect of connecting wires, calculate the magnitude of the e.m.f. induced in the loop. (2). Assuming that the resistances of the loop and connecting wires are hegligible, find the magnitude of the current through 100 (3). What is the direction of the current through 100 resistor? (form D to E or E to D )? Explain clearly how you arrived at the answer. (4). When the current flows through the loop a tension is developed in the wire, Explain how this arises and calculate this tension at the instant when the magnitude of the magnetic flux density, threading the loop is 0.1 T.

AL/ 2007 PHYSICS


01 SI unit of surface Tension is (1) 02. N (2) N m-1 (3) Nm (4) N m-2 (5) Nm2

Dimensions of a certain quantity when multiplied by [L]3 yield the dimensions of work. The physical quantity would be (1) Force (2) Momentum (3) Pressure (4) Mass (5) Velocity

03.

If the absolute temperature of a body is doubled, the rate at which the energy is radiated will (1). Remain the same. (4). Increase eight times. (2). Increase two times. (5). Increase sixteen times. (3). Increase four times.

04.

An e.m.f. is induced across the length of a wire when it is moving in a uniform magnetic filed. This e.m.f. dose not depend on (1) (4) Velocity of the wire (2) Radius of the wire (3) Length of the wire

Flux density of the magnetic field

(5) The angle that the wire makes with the magnetic field

05.

Consider the following statements made regarding the photoelectric effect. (A) (B) (C) This effect can be explained by assuming light as energy packets For a given incident monochromatic light, the energy of emitted electrons does not depend on the material. Rate of emission of electron depends on the intensity of the incident light.

Of the above statements, (1) (4) Only (A) and (B) are true (2) Only (B) and (C) are true (5) (3) Only (A) and (C) are true

All (A), (B) and (C) are true.

All (A), (B) and (C) are false.

06.

A sound emitted by a source of intensity I reaches a certain point. The change in the sound intensity level at the same point when the sound intensity by increased to 2I is (log 2 = 0.3) (1) 0.3 dB (2) 3 dB (3) 6 dB (4) 9 dB (5) 15 dB

07.

Consider the following statements made regarding a monochromatic light ray refracting through a glass prism placed in air. (A) (B) (C) The speed of the light ray inside the prim is lower than that outside the prism. The frequency of the light ray inside the prism is lower than that outside the prism. The wavelength of the light ray inside the prism is lower than that outside the prism.

Of the above statements, (1) (4) 08. Only (A) is true (2) Only (C) is true (5) (3) Only (A) and (B) are true

Only (A) and (C) are true

All (A) , (B) and (C) are true

Figures A, B, C and D shown four ways in which a light spring balance can be loaded with a weight of 100 N using frictionless pulleys.

The scale readings of the spring balance in the four cases would be A (1) (2) (3) (4) (5) 100 N 100 N 100 N 100 N 100 N B 100 N 0 100 N 0 100 N C 100 N 200 N 100 N 200 N 200 N D 100 N 100 N 200 N 200 N 200 N

09.

Consider the following statements made above the linear expansivity of a material. (A) (B) (C) Its SI unit is K-1 Its value changes when the temperature is measured in Celsius instead of Kelvin. Its value changes when the temperature is measured in Fahrenheit instead of Kelvin.

Of the above statements, (1) (4) 10. Only (A) is true (2) Only (A) and (C) are true (5) (3) Only (A) and (B) are true

Only (B) and (C) are true

All (A), (B) and (C) are true.

The variation of the speed u with time t of a simple harmonic oscillator is shown in the figure. The variation of its velocity u with time t is best represented by

11.

A normal eye ball has a diameter of 2 cm as shown in the figure. The magnitude of the minimum power of the eye lens is (1) 0 (2) 10 D (3) 25 D (4) 50 D (5) 100 D

12.

The size of the image of an object placed at a distance of 10 cm from a convex lens is twice that of the object. If the image is erect, the focal length of the lens is (1) 7 cm (2) 10 cm (3) 20 cm (4) 30 cm (5) 40 cm

13.

The focal length of the lens of a simple microscope is 10 cm. If the near point of an eye is 25 cm, the approximate value of the object distance required to obtain the maximum angular magnification is (1) 5 cm (2) 6 cm (3) 7 cm (4) 8 cm (5) 9 cm

14.

An object weighs 100 N on the earth surface. When it is carried to a height equal to the radius of the earth, from the earths surface , its weight becomes (1) 10 N (2) 25 N (3) 50 N (4) 75 N (5) 100 N

15.

A cylindrical capacitor is formed by inserting two sheet of paper of dielectric constant 4 and thickness 10-4 m, alternately between two rectangular sheets of metal foils, each of length 1 m and breadth 10-2 m, and rolling them as shown in the figure. (0 = 9 x 10-12 F m-1)

The capacitance of the capacitor is (1) (2) 3600 pF 360 pF (3) 36 pF (4) 18 pF (5) 3.6 pF

16.

The figure shows a spherical conducting shell. A point charge + Q is placed at the centre of the shell and a charge q is given to the shell. Finally the shell will have

(1) Zero charge on the inner surface, - q on the outer surface. (2) surface. (3) (4) (5) Q charge on the inner surface, - q on the outer Q charge on the inner surface, - q + Q on the outer surface. + Q charge on the inner surface, -q Q on the outer surface. Q q/2 on the inner surface, +Q q/2 on the outer surface.

17.

If a wire of resistance R and length l is used to form another wire of length 2l without changing its volume. the resistance of the new wire is (1) 4R (2) 3R (3) 2R (4) R (5) R/ 2

18.

Four identical electric bulbs are connected to a battery as shown in the figure. If all the bulbs are lit, and the intensities of the bulbs A, B and C are IA, IB and IC respectively then

(1) > IB = IC (4) IA > IB > IC = IB = IC

IA > IC > IB (3)

(2) IA IB > IC > IA (5) IA

19.

A potentiometer is balanced by connecting a cell of e.m.f. 2 V across A and B, as shown in the figure. The same balance length can be obtained if another cell E of suitable e.m.f. is connected in series with the 2 V cell as

20.

An archeologist extracted 100 mg of carbon from an ancient wooden tool and found that it is as radioactive as 100 mg of carbon extracted from a live tree. Half-life of carbon -14 is 5730 years. How old is the wooden tool ? (1) 1432.5 years (2) 5730 years 11460 years (5) 22920 years (3) 10162.5 years (4)

21. Consider the following statement made regarding the logic gate shown in the figure.

(A) (B) (C)

When P = 1, R = Q When Q = 0, R = P When P = 0, R = 0

Of the above statements, (1) Only (C) is true. (A) and (C) are true. (4) 22. (2) Only (A) and (B) are true. (5) (3) Only

Only (B) and (C) are true.

All (A) ,(B) and (C) are true.

A ball B is projected horizontally with speed u and a ball A is dropped vertically from rest at the same instant as shown in the figure. Which of the following statement is true ? ( Neglect air resistance)

(1) A reaches the ground first with a higher speed than B. (2) B reaches the ground first with a higher speed than A. (3) A reaches the ground first with a lower speed than B. (4) Both A and B reach the ground at the same instant with the same speed. (5) Both A and B reach the ground at the same instant but B with higher speed than A.

23.

As shown in the figure a ball of mass 6 kg released from rest at point A on a smooth track ABCD slip without rolling. The portion DE of the track is rough. If the ball climbs up to a vertical height of 3 m along the rough surface, the energy lost due to friction is

(1). 240 J

(2). 180 J

(3). 120 J

(4). 60 J

(5). 0

24.

The two uniform disks A and B shown in the figure are made of the same material and have equal masses. The radius of the disk A is greater than that of B. The disks are kept in isolation at outer space. Consider the following statements. (A) (B) (C) The disk A takes a longer time than B to gain a given speed under an external force acting through the centers of the disks. The disk B takes a longer time A to gain a give angular speed under an external torque about the axis of the disks. The disk B has a higher rotational inertia about the axis of the disk than disk A.

Of the above statements, (1) Only (A) is true. Only (B) and (C) are true. (4) 25. (2) Only (A), (B) and (C) are true (5) (3)

All (A), (B) and (C) are true.

All (A) , (B) and are false.

Figures A and B show two ways in which a painter could use a system consisting of a platform P, a pulley and a rope in painting tall building. The total weight of the painter and the platform is 400 N. If the rope is light then the tension of the rope in the two cases are A (1). 400 N (2). 400 N (3). 200 N (4). 200 N (5). 100 N B 400 N 200 N 400 N 200 N 200 N

26.

A trolley is moving with a constant velocity u. Three men , A, B, and C, are pulling three strings in such a way that their tension are T1 , T2 , and T3,

respectively, as shown in the figure. When the trolley moves a distance L, the work done by the men are A C (1) T3L (2) -T1L 0 (3) (4) (5) T1L T1L 0 -T2L T2L 0 0 0 0 T2L T1L T2L B

27.

A toy in the form of a child-figure holding a section a thin ring, which carries two identical heavy metal balls, is made from a thin metal sheet as shown in figure. If the toy can be balanced in stable equilibrium from the toe of the child-figure, most probably the centre of gravity of the system can be found close to a point (1) (3) P R (2) (4) Q S (5) T

of the

28.

Starting form rest, a sphere takes a time t to roll down a rough inclined plane. If the plane is made frictionless, the time taken by the sphere to slip down will be (1) (4) t (2) higher than t (3) determined by the mass of the sphere. (5) lower than t determined by the radius of the sphere.

29.

An organ pipe filled with O2 has a fundamental frequency 0. If the pipe is filled with H2 at the same temperature and pressure, the new fundamental frequency of the pipe is (relative molecular masses of H2 and O2 are 2 and 32 respectively),

(1) 30.

(2)

(3)

(4)

2 0

(5)

4 0

Rays from a monochromatic source of light are deviated by an optical element as shown in the figure. This optical element is likely to be (1) (2) (3) (4) (5) a convex lens. a concave lens. a single prism. a combination of two prisms. a combination of a prism and a convex lens.

31.

Figures A, B and C show wavefronts emitted from three sources of sound. These figures respectively represent sources

(1) (2) (3) (4) (5) 32.

Moving to the right, moving to the left, and stationary. Moving to the left, moving to the right, and stationary. Stationary, stationary, and moving to the right. Moving to the left, moving to right, and moving to the left with the speed of sound. Moving to the left, moving to the right, and moving to the right with the speed of sound.

A student vibrated a tuning fork and listened to its sound while keeping it in air. Then he vibrated this tuning fork again with the same amplitude and listened to the sound while holding against a large wooden board. (1) (2) (3) Sound intensity heard by him in both cases is the same. Sound intensity heard when the tuning fork is in air is larger than when it is held against the wooden board. The time during which the tuning fork goes on vibrating is the same in both cases.

(4) (5) 33.

The time during which the tuning fork goes on vibrating is higher when it is kept on the board than in air. The time during which the turning fork on vibrating is higher when it is kept in than on the board. air

A tuning fork is at resonance with a sonometer wire. Consider the following statements (A) (B) A standing wave is set up in the wire. If the tension of the wire is increased its resonance length will decrease.

(C) The amplitude of vibration would be maximum if it resonates in the fundamental mode of vibration. Of the above statements, (1) (4) 34. Only (C) is true. (2) Only (A) and (B) are true. (5) (3) Only (A) and (C) are true.

Only (B) and (C) are true.

all (A), (B) and (C) are true.

Which of the following statements is true for a mixture of ideal gasses at a given temperature ? (1) (2) (3) (4) (5) All the gas molecules in the mixture have the same speed. Molecules of each component of the gas mixture have the same average kinetic energy. Lighter gas molecules have a lower average kinetic energy. Heavier gas molecules have lower average kinetic energy. Root mean square velocities of gas molecules of each component of the gas mixture are the same.

35.

A volume V1 of air at 100% relative humidity is mixed with volume V2 of completely dry air at the same temperature and pressure so that the final volume becomes V1 + V2. The relative humidity of the mixture is

(1)

(2)

(3)

(1)

(5)

36.

Consider a situation where a layer of ice being formed on Arctic sea water due to a constant temperature difference between sea water and the atmosphere. The variation of the rate (R) at which the heat is extracted from a unit area of ice-atmosphere interface by the atmosphere with time (t) is best represented by

(1)

(2)

(3)

(4)

(5)

37.

A particle with change q and mass m travels perpendicular to a uniform magnetic filed along a circular path of radius R with frequency . The magnitude of the magnetic flux density is given by (1)

38.

When the value of R2 in the figure shown is varied from 0 to infinity, the corresponding variation of V with R2 is best represented by

(1) (2) (3) (4) (5)

4/3 5/3 7/4 6/5 2

40.

The cell E1 and E2 shown in the figure have zero internal resistance. The voltage V across the terminals A and B is (1) (4) E1 E2 ( E1 E2 )/2 (2) E1 + E2 (3) ( E1 + E2 )/ 4

(5) ( E1 + E2 ) /2

41.

The figure shown an operational amplifier circuit with a light dependent resistor (LDR) and a 1 K resistor. The supply voltage to the operational amplifier is 16.5 V and its saturation voltage is 15 V. The resistance of the LDR is 1 M at complete darkness and 100 at bright light. The approximate values of the output voltage of the circuit Vout at complete darkness and bright light will be respectively (1) (4) 1.5 V and 15 V - 1.5 V and -16.5 V (2) (5) 1.5 V and 16.5 V 1.5 mV and 15 V (3) -1.5 mV and -15 V

42.

In the circuit shown, the transistor operates in the active mode with V BE = 0.6 V. The collector-emitter voltage V CE in the circuit is approximately (1) (4) 0 6V (2) (5) 2V 10 V (3) 4V

43.

As shown in the figure, a device is made to measure the magnitude of a force by applying it to a uniform metal rob of length l and area of cross-section A, and measuring the resultant compression (l). E is the Youngs modulus of the material of the rod. The smallest value of the compression that can be measured with a measuring instrument attached to the rod is l0. If the smallest value of F that can be measured with the device is F0, then the length l of the rod should be such that

(1)

(2)

(3)

(4)

(5)

44.

As shown in figure A, a child sitting on a rotates chair, rotates with an angular speed .The moment of inertia of the system with the child around the axis of rotation is 2 kg m2 . As shown figure B, while rotating , the child catches a thin ring R of mass 4 Kg and diameter 1 m, which is falling vertically with its plane horizontal, and with no angular momentum. The final angular momentum of the whole system would be (1) 0 (2) ( 2/3) (4) (2/3) (5) (1/3) (3)

in

45.

A boat made of metal floats in water with one fifth of its volume submerged. If a second boat is made with a volume five times bigger than the first using the same mass of the same metal that has been used to construct the first boat, then the ratio,

(1)

(2)

(3)

(4)

(5)

10

46.

Two point charges +q1 and -q2 are placed as shown in the figure. Resultant electric filed intensity could be zero at a point (1) (2) (3) (4) (5) A, if q1 = q2 A, if q1 > q2 A, if q1 < q2 B, if q1 = q2 B, if q1 > q2

47.

A radioactive source is placed at the bottom of a hole in a lead block. The beam of radiation emanating through the hole is allowed to pass through a magnetic filed as shown in the figure. Three separated beams A, B and C could be, respectively (1) , - and , and , - and (4) (2) -, and , and (3) (5)

48.

A metal block is hung from a support by a string P as shown in the figure. An identical piece of string Q is attached underneath the block. Consider the following statements, (A) If Q is taut the tension in P is greater than that of Q.

(B) If Q is pulled with slowly increasing tension, then P has a tendency to break before Q. (C) If Q is pulled with a jerk, then Q has a tendency to break before P.

Of the above statements, (1) (4) Only (A) is true. (2) Only (A) and (B) are true (5) (3) Only (A) and (C)are true.

Only (B) and (C) are true.

All (A), (B) and (C ) are true.

49.

A decoration consists of four independently rotating sets of small lanterns P, Q, R and S which are fixed to a rotating central pole as shown in the figure (A). All the rotations take place around vertical axes. Which of the following modes of rotation provides the best stability to the entire decoration ?

50.

A system of coplanar forces OA, OB, OC, OD, OE and OF acts on an object as shown in the figure. Magnitude of OA = 2 OD and OE = 2 OB. The resultant force on the object is most likely to be (1) (2) Along the direction of OC. Along the direction of OE.

(3) OA.

Along the direction of OF. (5)

(4) Zero

Along the direction of

51.

Water drain through an opening of area A1 in a container of crosssectional area A2 as shown in the figure. If the motion of the water surface in the container is not ignored the speed u at which the water drains is given by

52.

A uniform electric filed of magnitude 400 V m-1 is acting in the direction as shown in the figure. If VA and VB are the electric potentials at points A and B respectively, then VB VA is equal to (1) (4) 6V 3V (2) (5) 3V 6V (3) 0

53.

The internal resistance of the battery shown in the circuit is zero. The value of R0 is adjusted until the voltage across it becomes 5 V. The equivalent resistance of the part of the network to the right of XY is

(1) R0 (R0+R1) (5) 54.

(2) R1

R0 + R1

(3)

( R0R1 ) / (R1-R0 )

(4)

( R0R1) /

Three identical circular wire loop are placed perpendicularly to uniform magnetic fields of flux density B. The extent of the magnetic field are different form one another in situations P, Q and R as shown in figures. The extent of the magnetic field in Q is equal to the area of the loop. When the flux density B varies with time at the same constant rate, the induced e.m.f.s of the respective wire loop are EP, EQ and ER. Which of the following is true regarding the magnitude of EP, EQ and ER ? (1) (3) (5) EP = 0, EQ = ER EP = EQ = 0, ER 0 EP < EQ < ER (2) (4) EP = 0, ER > EQ EP < EQ, EQ = ER

55.

A rectangular wire frame made from a smooth resistive wire is connected to a battery of e.m.f. E with negligible internal resistance as shown in the figure. XY is a pipe cut from the same wire , which can slide along the wire frame. A non conducting liquid film of surface tension T is formed in the region CDXY, and the entire structure is placed in a uniform magnetic field of flux density B in the direction shown. If XY = XD = CD = CY, and the current through AX is l, then the wire XY will tend to move to the right when (1). B > aaaaa (2). B > aaaaa (3). B< aaaa (4). B > aaaa (5). B < aaaa

56.

An ideal gas taken through a thermodynamic cycle as shown in the figure. Process 12 is isothermal and during the process 60 J of heat enters the system. Process 2 3 takes place at constant volume and during this process 40 J of heat leaves the system. The change in internal energy (U) of the system during process 3 1 is (1). 40 J (2). 20 J (3). 0 (4). + 20 J (5). +40 J

57.

Thermometers must possess good sensitively as well as a good accuracy. In this connection which of the following is true regarding a mercury in-glass thermometer ?

58.

Two bulbs, A (110 V, 40W) and B (110 V, 100W) are connected in series with an electric supply of 220 V. Which of the following statements is false ?

(1). The current through A is the same as the current through B. (2). The potential drop across A is greater than the potential drop across B. (3). The current through B is less than its rated current. (4). The power dissipation of A is greater than the power dissipation of B. (5). There is a higher probability of burning the bulb B. 59. Figure shown the cooling curve of liquid wax of mass m, specific heat capacity S1 and latent heat of fusion L. The heat capacity of the container can be neglected. Consider the following statements. (A). Gradient of the curve AA at A is equal to the gradient of the curve BB at B. (B). The rate of heat released to the surrounding during the time T is mL/T. (C). Gradient of the curve AA at A = aaaaaaaa Of the above statements, (1). Only (A) is true. (2). Only (A) and (B) are true. (3). Only (B) and (C) are true.

(4). All (A) ,(B) and (C) are true. 60.

(5). All (A) ,(B) and (C) are false.

A tiny sphere with a static charge +q starts to fall through air under gravity at t = 0. After the sphere has reached terminal velocity, a vertically upward electric field E of constant magnitude is applied. A short time after the sphere changes direction of its motion , the electric field is removed. The variation of the velocity (u)

of the sphere with time (t) is best represented by

AL/ 2012 PHYSICS ( part B essay )


05. In this, you will investigate a few basic movements of a robotic arm shown in figure (1). The arm segments A and B of the robot have the ability to rotate in either direction around joints 1 and 2 in horizontal panels. Joint 3 allows segment C to move up and down. All three joints are operated by electric motors. Assume that only one movement around or across a joint is allowed at a given time and that there is no friction in any of the joints. (a). First consider an upward motion of segment C. This motion is described by the velocity () time (t) graph in figure (2). Mass of segment C is 0.1 kg . (1). (2). Calculate the acceleration of segment C during the first 2 seconds. The force acting on C are its weight, and the force applied by the motor for the motion of C. Calculate the force applied by the motor during the first 2 seconds. What are the magnitude and direction of the force applied by the motor on C during the last 2 seconds of motion? Suppose the magnitude of the maximum force the motor can exert on C is 1.2 N. If staring rest, C from moves up under this maximum force for 0.5 s, how far will it move ?

(3).

(4).

(b). Next consider a rotation of segment B (together with segment C ) occurring around joint 2. The angular velocity ()- time (t) graph in figure (3) shown this rotation. Assume that segment A is held fixed during this rotational motion. The moment of inertial of the combined system

of segments B and C around the axis of joint 2 is 0.01 kg m2. (1). Calculate the torque applied by the motor on B during the first 4 seconds of motion in figure (3). (2). Calculate the angular displacement of B during the 8s period shown in figure (3). (3). If the magnitude of the maximum torque that can be applied by the motor is 0.002 N m , what is the minimum time that will take for B to start from rest and come to rest again after an angular displacement of 3.2 radians ? (c). Now if segment A is allowed to rotate freely around joint 1, what would be the direction of rotation of segment A. when segment B, starting from rest, rotates clockwise around joint 2 ? Given reasons for your answer.

06.

Read the following passage and answer the questions given below. The Doppler effect for sound waves depends on three velocities, namely the velocities of sound, the source, and the observer will respect to the air. Normally air is considered to be stationary relative to the ground and therefore these velocities can be measured relative to be ground. However, this is not the situation with regard to light waves. Light as well as other electromagnetic waves require no medium, and they are capable of travelling even through a vacuum. The Doppler effect for light waves depends on two velocities, namely the velocity of light (c) and the relative velocity () between the source and the observer, as measured from the reference frame of either source or the observer. If a certain light source is at rest relative to us, we would detect light from it with the same frequency (0) as that of the source, and it is known as the proper frequency. If it is moving away from us with a speed ( << c), then the light we detect has a frequency that is shifted from 0 due to the Doppler effect and is given by the following

formula.

= 0 (1 ) Where = /c

However , measurements involving light are usually made in wavelengths rather than frequencies, and the above formula can be rewritten in terms of wavelengths in the following from. = /0 c where = 0 The quantity is called the Doppler shift. If the light source is moving away from us, is longer than 0, is positive, and the Doppler shift is called a red shift. If the light source is moving toward us, then is shorter than 0, is negative, and the Doppler shift is called a blue shift. Using astronomical observations of stars, galaxies and other sources of light, scientists can determine how fast the sources are moving , either directly away from us or directly towards us by measuring the Doppler shift to the light that reaches us. Two regions of interstellar gas orbiting the core of a galaxy known as M87 at a radius r = 100 light years is shown in figure (1). One region is moving towards us with a speed and the other region is moving away from us with the same speed. Figure (2) shows the variation of intensity (I) with wavelength () of light reaching us from those two region. The gas is under the influence of the gravitation force due to the mass M of the core of the galaxy. This mass of core is about two billion times the mass of our sun, strongly suggesting that a super massive black hole occupies the core. (a). (1). Doppler effect for sound waves depends on three velocities. Name them. (2). These velocities are normally measured relative to the ground. What is the reason for this ? (b). Why does the Doppler effect for light depends only velocities ?

(c). Starting from = 0 (1 ), derive the relationship = /0 c. [ Hint: When << 1, aaa =1 + ] (d). (1). From figure (2), determine the values of two wavelengths at which the intensities are peaked. (2). Which peak corresponds to the gas moving towards us ? (3). If the gas were not moving relative to the core, What is the wavelength 0 (proper wavelength) of the light that would be by us ? (4). What is the Doppler shift () of the light from the gas moving away from us ? (5). Hence determine the speed of the gas. Round off your answer to the nearest integer ( c = 3.0 x 108 m s-1). (6). Is << 1 ? Justify your answer. 10
-11

(e). (1). Determine the mass M of the core of the galaxy. (G = 6.0 x N m kg-2)
2

(2). What is believed to be occupying the core of the galaxy ? 07. Figure (1) shows the stress-strain curve for a uniform steel rob. Identify the points A, B and C. An underground storage (S) of length 150 m, and width 6 m is to be constructed at a depth of 20 m from the ground level. Figure (2) shows the side view and figure (3) shows the front view of the storage . The weight of the soil existing above the roof of the storage is to be supported entirely by 30 cm x 30 cm square steel columns (C). The soil has a uniform density of 3.0 x 103 kg m-3

(1). Calculate the total weight of the soil that the columns must support.
(a)

(2). When is the number of columns needed to keep the compressive stress on each column at 2 x 108 N m-2? Assume that the weight of the soil is equally distributed among the columns. Neglect the mass of the roofing material. (b). (1). Determine the Youngs modulus of steel from the curve given in figure (1) above. (2). If the height of a steel column is 4.995 m what was its original uncompressed height? (c). If the columns have a circular cross-section of radius 15 cm instead of the square cross-section of 30 cm x 30 cm mentioned above, does the number of columns calculated in (a) (2) above would be less, same or more ? Given reasons for your answer.
08. Two metal plates A and B kept parallel to each other in a vacuum are connected to a voltage source as shown in figure (1). A molecular ion of mass m and charge +q , starting from rest the plate A accelerates towards the metal plate B under the influence of the voltage V maintained between the plates. (a). (1). Write down an expression for the kinetic energy gained by the ion when it reaches the plate B. (2). Derive an expression for the velocity acquired by the ion when it reaches the plate B. (3). If d0 is the distance between the plates derive an expression for the time (t) taken by the molecular ion to reach the plate B. (b). Suppose the metal plate B is now replaced with a

wire mesh so that the ions moving through the region AB could enter a field free region and move towards an ion detector D placed at a distance S from the mesh B as shown in figure (2). Consider two molecular ions 1 and 2 of mass m and change +q suddenly being formed at time t = 0 at distances d1 and d2 from the wire mesh B as shown in figure (2). If they start from rest move towards B under the electric field (1) Derive expression for times t1 and t2 taken by the ions 1 and 2 to reach the mesh B. and Indicate which ion reaches the mesh first

(2) Derive expression for velocities 1 and 2 of ions 1 and 2 when they reach the mesh B. Indicate which ion has the higher velocity when they reach B. (3) Derive an expression in terms of t1, t2, 1 and 2 for the suitable value for the distance S at which the detector D has to be placed so that it detects both ions 1 and 2 at the same time as shown in figure (3).

9.

Answer either part (A) or part (B) only. (A). (a) Figure (1) shows a circuit powers by a 12 V battery with negligible internal resistance. The two bulbs A and B are rate at 3 V, 0.1 A and 12 V, 2 A respectively. C and D are two devices having internal resistance 6 each. (1). Calculate the value of resistor R1 that would provide the rate voltage to bulb A. (2). Calculate the voltage across C and the power dissipated in the 10 resistor. (3). In order to be able to limit the current through D between 0.5 A and 2 A, what should be the value of the variable resistor R2? (4). Suppose three fuses with current ratings 4 A, 5 A and 10 A are given. In order to make it possible to operate all devices simultaneously, under the above conditions, which fuse would be most suitable to be connected to this circuit ? (b) Electrical circuits such as the one above are constructed by mounting electrical components on insulated boards, and joining the terminals of the components by copper wires. In modern circuits, however, such connections are made by thin copper strips printed on insulated boards.

A part of a printed circuit board is shown in figure (2), and an enlarge diagram of one copper strips is shown in figure (3). For all calculations below, take the thickness of copper strip, h, as 0.3 mm. (1). Calculations the resistance of a 10 cm long copper strip of width w = 1 mm. (Resistivity of copper is 1.8 x 10-8 m.) (2). Calculate the voltage across this strip and its power dissipation, when a current of 0.1 A passes through it. (3). If all the heat dissipated in one second is accumulated in the strip without being lost to the environment, what will be its increase in tempeureter ? (Specific heat capacity and density of copper are 400 J kg1 K-1 and 9 x103 kg m-3 respectively.) (4). Copper strip carrying large currents are normally made wider than those carrying small currents. Give two reasons for this. (B). (a). Write down the truth for a 2- input AND gate. Use symbols A and B for inputs and F for output.

(b). The block diagram of the circuit shown in figure (1) is given in figure (1). (1). Write down the truth table for the circuit shown in figure (1).

(2). Hence, show that the circuit shown in figure (1) operates as follows: F0 = 1 only when A = 0 and B = 0, and F0 = 0 otherwise. (c). Suppose now you use a 3-input AND gate in the circuit shown in figure (1) above instead of a 2-input AND gate. Let the third input be E. then the block diagram will take the from shown in figure (3). (1). Draw the circuit diagram corresponding to the block diagram in figure. (2). By filling the two truth tables shown, show that the circuit will operate similar to the circuit given in figure (1) when E = 1, and the output F0 = 0 when E = 0 irrespective of the values of A and B. (d). Now draw a circuit diagram using a 3-input AND gate one NOT gate to operate as follows. The output F1 = 1 only when A = 0, B = 1 and E = 1 F1 = 0 when E=0 (e). Similarly draw two separate circuits using 3-inut AND gates and NOT gates, to operate as follows. (1). Output F2 = 1 only when A = 1 and B = 0 and E =1 F1 = 0 when E = 0 (2). Output F3 = 0 only when A = 1 and B = 0 and E = 1 F3 = 0 when E = 0 (f). Now combine all four circuits draw under (c) (2), (d), (e) (1) and (e) (2) and draw a

single circuit so that it will have only 3 common inputs A, B and E, and four outputs F0, F1, F2 and F3.

The circuit that you have draw should conform with the block diagram given in figure (4). (g). Suppose you are given four devices, an electric fan, an electric kettle and an electric motor which can be switched ON or OFF with logic signals 1 or 0 respectively. (1). Draw a block diagram to how you would connect the devices shown in figure (5) to the block diagram given in figure (4) so

that any one of them can be selected and operated, one at a time. Write down the combination of appropriate logic signals that you would apply to the inputs A and B to select each device. (2). How would you keep all non operative condition using logic signals ? 10. Answer either part (A) or part (B) only. (A). (a). Consider a pond of cross section 2 m x 2 m, and containing pure water constantly being exposed to direct sunlight. (see figure 1) The amount of solar heat radiation falling on the pond is 100 W m-2 and assume that its is constant for the calculations below. Furthermore assume that solar heat is incident normal to the water surface at all times, no heat transfer occurs between water and the walls of the pond and that no heat is absorbed by water directly from sunlight. All the heat is absorbed by a blackened metal sheet placed at the bottom of the pond and then transferred to water near the bottom by conduction. If the amount of heat absorbed by the metal sheet over a 7 minute period entirely contributed to raise the temperature of a thin layer of water of mass 40 kg just above the metal sheet, how much will be the temperature rise in water? (Take specific heat capacity of water as 4 200 J kg-1 K-1 )
(1)

Let densities of water at 0 0C and at 0C be 0 and respectively. Obtain an expression for in terms of 0, and the volume expansively of water .
(2)

(3) explain why convection currents will occur when water is heated as mentioned in (a) (1) above. (b). A solar is a pond used to collect store solar energy as heat. Solar heat reaching the bottom of such a pond is trapped by suppressing convection current.

A very simple model of a solar pond with a 2m x 2m area is shown in figure (2). It has three distinct layers. The top layer has relatively pure water. The bottom layer has a very high salt concentration resulting a high density. The density is uniform throughout that layer. In the middle layer, the salt concentration and density decreases gradually with height.

For the following parts, assume that the initial temperature of water throughout the pond is 30 0C.
(1) In a practical solar pond, the temperature of the bottom layer can

reach about 90 0C.If the mass of water in this layer is 6000 kg and if it receives heat radiation at the constant rate of 1000 W m2 , how long will water to take to reach 90 0C? Assume that this heat is entirely used to increase the temperature of water, and that salt water has the same specific heat capacity as pure water.
(2) Taking 0 = 1554 kg m-3 for salt water, calculate the density of salt

water at 90 0C.(Volume expansively of salt water is 4 x 10-4 K-1)


(3) If the top layer remains at 30 0C , can there be convection

current from the bottom to the top layer under the above condition ? Justify your answer. (Take density of pure water at 30 0C as 1000 kg m-3)
(4)

(1). When the temperature of the bottom layer increase from 30 0C to 90 0C, calculate the amount of heat stored in that layer.

(2). Suggest a method to use this energy for a practical purpose. (5) In a practical solar pond, heat loss through the walls must be minimized. If a Styrofoam layer of thickness 10cm is used as an insulation between water and walls of the pond, and if the temperature of the walls stays at 40 0C while water is at 90 0C ,

what will be the rate of heat loss per m2 (Heat conductivity is 0.01 Wm-1 K-1)

through Styrofoam?

(B). In 1924 Louis be Broglie proposed that a particle having a linear momentum p can be described by a matter wave known as a de Broglie wave. (a). (1). Write down an expression for the de Broglie wavelength (), in term of the Planck constant h and p.

(2). For a particle of mass m and kinetic energy E, rewrite the above expression in terms of h, m and E. (b) (1). (2). A vessel is filled with helium gas at temperature T and atmospheric pressure of 105 Pa. Write down an expression for the mean kinetic energy E of helium atoms in the Boltzmann constant k and T. Using the expression derived in (a) (2) above write down an expression for the mean de Broglie wavelength of helium atoms in terms of h, k, T and mass m of a helium atom. Calculate at T = 27 0C. (The numerical values of the constants are given at the end of the question.) [Take aaa = 3] If a is the mean distance between helium atoms, by taking the total volume of helium gas to be Na3, where N is the number of helium atoms present in the vessel, determine a. Consider helium to be an ideal gas. [Take aaa = 4]. Can the helium atoms be treated as particles under these conditions ? Given reason for your answer. If the volume of the gas could be decreased without changing its pressure by cooling it down , at a certain temperature T the mean de Broglie wavelength of helium atoms can be made equal to be made equal to the mean distance between helium atoms. Derive an expression for T, in term of h, m and k. (Planck constant h = 6.6 x 10-34 J s; Mass of a helium atom m = 6.0 x 10-27 kg; Boltzmann constant k = 1.4 x 10-23 J K-1)

(3). (4).

(5). (6).

AL/ 2012 PHYSICS ( M.C.Q)


1. Which of the following dose not represent a fundamental unit in the SI system? (1). m (2). N (3). Kg (4). S (5). K 2. If the distance between two masses is doubled the gravitational force between them will decrease by a factor of (1). 2 (2). 4 (3). 6 (4). 8 (5). 12 3. Figure shown a thin uniform L-shaped metal sheet. The centre of gravity of the sheet is most likely to be found at the point (1). A (2). B (3). C (4). D (5). E

4. The minimum amount of work that be done in order to fix a light elastic string of initial length l0 between two parallel walls separated by a distance d (d > l0) with a tension T is (1). Aaaaaaaa (2). aaaaaa (3). Aaaaaa (4).aaaaaaaa (5).aaaaaaaa 5. A vessel contain an ideal gas at 27 0C. If the temperature of the gas is increased
to 127 0C, the ratio, aaaaaaaaaaaaaaaaaaaaaaaaaaaaaaaaaaaaaaaaaaaaaaaaaaaaaa will become (1). 127/27 (2). 16/9 (3). 4/3 (4). 3/4 (5). 27/ 127

6. The mass of body A is twice that of body the material of body A is three times that with equal amounts of heat. If the body A change of T, then body B will experience T/2 (2). 2/3 T (3). T

B. The specific heat capacity of of body B. They are supplied experiences a temperature a temperature change of (1). (4). 3/2T (5). 6T

7. Consider the following statements made about laser light. (A). The energy of a photon in a laser beam of certain frequency is higher than the energy of a photon of the same frequency in a normal light beam. (B). A laser beam cannot be refracted by a glass prism. (C). All the photons in a laser beam have the same energy, the same phase, and the same direction. Of the above statements, (1). Only (B) is true. (2). Only (C) is true. (3). Only (A) and (B) are true. (4). Only (B) and (C) are true. (5). All (A), (B) and (C) are true. 8. A noisy workplace has a noise level of 90 dB. This was reduced to a less uncomfortable level of 70 dB. The ratio aaaaaaaaaaaaaaaaaaaaaaaaaaa is equal to (1). 0.9 (2). 0.5 (3). 0.1 (4). 0.01 (5). 0.001 9. A monochromatic ray of light is incident no a glass prism and suffers minimum deviation while going through the prism. The emergent ray most likely will go through the point (1). A (2). B (3). C (4). D (5). E 10. Which of the following statements made about electric field lines is false? (1). Electric field lines can be either straight or curved. (2). Electric field lines can be parallel to

one another. (3). Electric field lines can from closed loops. (4). Electric field lines begin on positive charges and end on negative charges. (5). Electric field lines can never interest with one another. 11. A spherical Gaussian surface surrounds a point charge q. The following changes were made to the system. (A). The magnitude of the charge was tripled. (B). The radius of the spherical Gaussian surface was doubled. (C). The spherical Gaussian surface was changed to a surface of a cube. (D). The charge was moved to another location inside the surface. Of the changes mentioned above, the net electric flux through the surface is changed only in (1). (A). (2). (A) and (B). (3). (C) and (D) (4). (A), (B) and (D). (5). All (A), (B), (C) and (D). 12. An ideal transformer operates at V p = 12.0 kV ac on the primary side and supplies electricity to a number of nearby houses at Vs = 240 V, The turns ratio, number of turns in the primary of the transformer. number of turns in the secondary (1). 0.02 (2). 0.2 (3). 25 (4). 50 (5). 100 13. Two copper wires have the same volume, but wire 2 is 20% longer than wire 1. The ratio, resistance of the wire 2 is resistance of the wire 1 (1). 0.83 (2). 0.91 (3). 1.11 (4). 1.20 (5). 1.44

14. A water layer exists between the bottom of a cylindrical bottle and a glass plate as shown in the figure. The radius of the bottom of the bottle is r. When the bottle is raised slowly, at one instant the contact angle between water and the bottom of the bottle becomes . (see figure) The magnitude of the force on the bottom of the bottle at that instant due to surface tension T of water , is (1). 2rTsin (2). 2rTcos (3). 2 2 2r Tsin (4). r Tcos (5).4rTsin

15. Which of the following is not true regarding the rate at which a body emits radiant energy? (1). It is proportional to the surface area of the body. (2). It is proportional to the 4th power of the absolute temperature of the body. (3). It is proportional to the emissivity of the surface of the body. (4). It depends on the temperature of the surrounding. (5). It dose not depend on the thermal capacity of the body. 16. The graph shows the variation of the maximum kinetic energy (K max) of emitted photo-electrons from a metal with the frequency () of the incident radiation. The work function of the metal is (1). 6.0 e V (2). 4.0 e V (3). 2.5 e V (4). 2.0 e V (5). 1.0 e V 17. A radioactive isotope of iodine, 13153 I decays to particle is emitted in this decay? (1). (2).(5). n
131 54

X e. . What type of (3).+ (4) p

18. Consider the following statements made about the information that can be obtained from dimensional analysis. (A). Number values of constants of proportional that the may appear in a physical equation can be determined by dimensional analysis. (B). Numerical sings of constants of proportional that may appear in a physical equation can be determined by dimensional analysis. (C). The units of constant of proportional that may appear in a physical equation can be determined by dimensional analysis. Of the above statements, (1). Only (A) is true. (2). Only (B) is true. (3). Only (C) is true. (4). Only (B) and (C) are true. (5). All (A), (B) and (C) are true. 19. Equal masses of three liquids of densities d1, d2 and d3 are added together. If the liquids mix together without causing any change, then the density of the composite liquid will be

aaaaaaaaaaaaaaaaaaaaaaaaaaaaaaaaaaaaaaaaaaaaaaaaaaaaaaaaaaaaaaaaaaa aaaaaaaaaaaaaaaaaaaaaaaaaaaaaaaaaaaaaaaaaaaaaaaaaaaaaaaaaaaaaaaaaaa aaaaaaaaaaaaaaaaaaaaaaaaaaaaaaaaaaaaa 20. A ball of mass 0.5 kg which is initially at rest, is struck by a bat. The variation of the force (F) on the ball with time (t) is shown in figure. The speed of the ball when it leaves the bat is (1). 10 m s-1 (2). -1 -1 -1 -1 8ms (3). 6 m s (4). 4 m s (5). 2 m s 21. Two small sphere A and B of putty of mass m and 3m respectively are suspended from a ceiling by mean of strings of equal length. Sphere A is drawn aside so that it is raised to a height h as shown, and then released. Sphere A collides with sphere B which is at rest , and they stick together. The maximum height to which the composite body swings is (1). 1/6h (2). 1/8 h (3). 1/4 h (4). 1/3 h (5). 1/2 h curvature r in a is the > aaaaaaa (5). >

22. A car of mass m man oeuvres a circular bend of radius of horizontal flat road with a speed . If the car skids then ( coefficient of friction between the road and a tire). (1). (2). < aaaaaaa (3). > aaaaaa (4). < aaaaaaaa aaaaaaaa

23. Photographs of an object that starts falling freely from rest at t = 0 are taken by a camera, first at t = 0 , and thereafter at the end of each second. Which of the following diagrams correctly indicates the location of the object at the end of each second? The vertical axes of the diagrams represent the distance (d) travelled by the object.

24. A satellite (S) moves in an elliptical orbit about the earth (E). If the speed of the satellite at point A is , then its speed at point B will be (1). /8 (2). /4 (3). (4). 2 (5). 4 25. Consider the following statements made about a particle of mass m attached to a light spring and performing simple harmonic motion as shown in figure. (A). The acceleration of the particle is always towards the center of motion. (B). The force on the particle is proportional to the square of the displacement from the centre. (C). The period of oscillation depends on the mass of the particle. Of the above statements, (1). Only (A) is true. are true. All (A), (B) and (C) are true. (2). Only (C) is true (3). Only (A) and (B) (4). Only (A) and (C) are true. (5).

26. Consider two rays moving towards a thin converging lens as shown in figure. After passing through the, two rays are most likely to be met at the point (1). A (2). B (3). C (4). D (5). E 27. Figure shown the changes occurred to a wave from of a monochromatic ray of light travelling in air (A) when incident normally and transmitted through a transparent medium (T). The refractive index of the transparent medium is (1). 1.5 (2). 2.0 (3). 2.5 (4). 3.0 (5). 3.5

28. The human tract (larynx) can be considered as a tube that is open at one end. If the length of this tube is 17 cm, the frequencies of the lowest two harmonics produced are given by (The speed of sound in air = 340 m s-1 ) (1). 500 Hz, 1500 Hz (5). 1500 Hz, 2500 Hz 29. A train travelling at a constant velocity while continuously sounding its horn with a frequency 0 moves towards an observer standing on a platform and then moves away from him. The variation of the frequency (2). 500 Hz, 1000 Hz (3). 1000 Hz, 2000 Hz (4). 100 Hz, 3000 Hz

() of the horn as heard by the observer with time (t) is best represented by

30. The graph shown the variation of a quantity y with another quantity x. Consider the following statements. (A).If the graph represents a wave travelling in a stretched string, along the x direction, y could be the displacement of a particle of the string, in a direction perpendicular to the motion of the wave, at a given instant. (B). If the graph represents a wave travelling in water, x could be the time and y cold be the displacement of a water molecule along the direction of the wave. (C). If the graph represents a vibration of a tuning fork, x could b e the time and y could be the velocity of the tip of one prong of the fork. Of the above statements, (1). Only (A) is true (2). Only (C) is true. (3). Only (A), and (C) are true. (4). Only (B) and (C) are true. (5). Only all (A), (B) and (C) are true. 31. A planet is observed by an astronomical telescope in normal adjustment, having an objective of focal length 14 m and an eyepiece of focal length 2 cm. Consider the following statements. (A). The distance between the objective and the eye piece is 1402 cm. (B). Angular magnification of the planet is 700. (C). The image of the planet is formed at the near point of the near point of the observer. Of the above statements, (1). Only (A) and (B) are true. (2). Only (A) and (C) are true. (3). Only (B) and (C) are true. (4). Only (B) is true (5). All (A), (B) and (C) are true. 32. Consider a process where air is quickly leaking out of a balloon. Which of the following is true for this process? Q W U

(1). (2). (3). (5).

+ _ 0 (4). 0

+ _ 0 0 + _

+ _ 0 _ _

33. The figure indicates the face and interface temperatures of a lagged composite slab consisting of four materials A, B, C and D of identical thickness and surface area through which the heat transfer is steady. If k A, k B, k C, and k D, are the thermal conductivities of materials A, B, C and D respectively then (1). K A > K B > K C > K D (2). K A < K B < K C < K D (3). K B = K D < K A < K C (4). K
B

= K D< K

<K

(5). K

= K D= K

>K

34. Consider the following statements made about the capability of a given thermometer to produce an accurate value for a temperature measurement. (A). In situation where quick change of temperature with time have to be measured, the given thermometer must be a one having large variation of the thermometric property with temperature. (B). Thermal capacity of the thermometer must be negligible when compared to the thermal capacity of the environment of which the temperature is measured. (C).Thermometric property must have a linear variation with the temperature. Of the above statements (1). Only (B) is true. (2). Only (A) and (B) are true. (3). Only (B) and (C) are true. (4). Only (A) and (C) are true. (5). All (A), (B) and (C) are true. 35. A light, conducting loop is suspended freely, and a half of the loop is inserted into a magnetic as shown in the figure. If the magnetic field begins to increase rapidly in strength, (1). The loop begins to move in the direction of the magnetic field.

(2). The loop begins to move against the direction of the magnetic field. (3). The loop begins to move (to the left) into the field. (4). The loop begins to move (to the right) out of the field. (5). The loop does not move at all. 36. Current through the 10 resistor is (1). 0 (2). 1.5 A (3). 3.0 A (4). 5.0 A 6.0 A (5).

37. A metal wire has resistances R1 and R2 at temperature 1 and 2 respectively. The temperature coefficient of resistivity of the metal, is given by (1). (1 2) (2). (R1 R2) (3). (R1 R2) (4). (R1 R2) (5). (R21 R12) (R1 R2) (12) (1 2) (R1 + R2) (R21 R12) (R1 R2) 38. Which of the following connection will have to be made in order to operate the transistor (Si) circuit given in the figure as a common emitter amplifier ? (1). XE, YB, AP, BQ, SE (2). PA, YE, XP, BS, QE (3). SB, YA, AQ, BQ, SE (4). XE, YB, AQ, BP, SA (5). YA, XE, AP, BS, QE 39.

The inverting input of a 741 operational amplifier operating with 10 V power supply voltage is provided with a voltage signal which increases

linearly with time (t) as shown in figure. The non-inverting input is with a rectangular voltage waveform of amplitude 5 V as shown. The output waveform of the operation amplifier is best represented by

40. Which of the logic circuits shown will operate in the following manner ? When S = 0, output F = X (value of X can be either 1 or 0) When S = 1, output F = 0 ( irrespective of the value of X)

(1). (A) only (2). (B) only (5). (B) and (C) only.

(3). (C) only

(4). (A) and (B) only

41. A large metal sheet bent into the shape shown in the figure is kept upright on the ground by means of four stretched ropes fixed to the ground. Magnitudes of the tensions in all ropes, TA, TB, TC and TD in still air are equal. When wind blows through the sheet in the direction XY (1). TA < TB and TD < TC (2). TA > TB and TD > TC (3). TA = TB and TC = TD (4). TA > TB and TC > TD (5). TA < TB and TC < TD

42. The variation of the velocity () with time (t) of a particle is shown in the figure. The corresponding displacement (s) time (t) curve is best represented by 43. A grain of sand is stuck to a tire of a vehicle at a distance r from its center. The radius of the tire is R. When the tire is rotating at an angular speed of , the sand grain detaches suddenly from the tire. If the air resistance is neglected, the horizontal component of the velocity of the grain relative to the vehicle immediately after detachment could have a value between (1). 0 and (R r) . (2). 0 and (r + R). (3). 0 and r . (4). r and r . (5). (R r) and (r + R). 44. A lead ball of radius a is fired from a toy gun in water in a large swimming pool as shown in the figure. The densities of water and lead are w and p b respectively and the viscosity of water is . If the x and y components of the velocity of the ball at a certain instant are x and y respectively then the magnitudes of the corresponding acceleration components at that instant would be x (horizontal) y (vertical) (1).aaaaaaaaaaaaaaa aaaaaaaaaaaaaaaaa

45. Water is found to condense on the outer surface of a cooled glass bottle of soft drink when kept in the atmosphere. The total amount of water condensed before it reaches the atmospheric temperature will not depend on (1). Initial temperature of the cooled bottle of soft drink. (2). Thermal capacity of the bottle of soft drink. (3). Rate of increase of temperature of the bottle of soft drink.

(4). Dew point of the atmosphere. (5). The thermal conductivity of glass. 46. Small amounts of water and ice of identical masses are placed in a thermally insulated container and allowed to come to thermal equilibrium. The variation of the temperature () of water and ice are recorded with time (t) and are shown in the same graph. Which of the following conclusions can be drawn about the behavior of water and ice from the given graph ? (1). Water has fully frozen and no ice has melted. (2). Water has partly frozen and ice has melted. (3). Water has partly frozen and ice has fully melted. (4). Water has fully frozen and ice has fully melted. (5). Water has fully frozen and ice has partly melted. 47.

Three identical wire loops A, B and C are placed in uniform magnetic fields as shown in figure. Magnetic fields are either increasing or decreasing at the same rate. If i1. i2 and i3 are the magnitudes of the induced currents in loop A, B, and C respectively then (1). i1 > i2 > i3 (2). i1 < i2 < i3 (3). i1 = i2 = i3 (4). i1 = i2 ; i3 = 0 (5). i1 = i2 = i3 = 0 48. A fuel-gauge in a vehicle uses a parallel plate capacitor made of two rectangular metal plates to determine the height of the fuel level in the tank. Each of the metal plates (ABCD and PQRS) has a with w and a height l. The height of the fuel level between the plates is h . (see figure) Appropriate electronic circuitry E determines the effective capacitance of the combined air and fuel capacitors. The effective capacitance of this system is given by (k = dielectric constant of fuel)

49.

A current carrying wire loop in figure (1) lies in the plane of the paper and consists of two concentric semicircles of radii 2R and R and two radial lengths. The smaller semicircle is bent out of the plane gradually until the loop is flipped over and lies entirely on the same plane again as shown in figure (2). An intermediate situation of the system when the loop is bent through an angle is shown in figure (3). The variation of the component of the magnetic flux density (B) directed into the page at the center (O) of the loop with angle is best represented by

50. In the circuit shown PQ is a variable resistor of 1000 , and the resistance between the terminals P and X varies linearly as terminal X moves from P to Q. As the terminal X moves from P to Q the variation of the ammeter reading (I) is best represented by

You might also like